\( \lim_n (n+1) \int_0^1 t^n f(t) \, dt = 0 \Longleftrightarrow \lim_{r \to 1^-} \frac{1}{1-r} \int_r^1 f(t) \, dt =0\)

Messaggioda Delirium » 01/02/2018, 15:15

Problema. Per \( f \in L^\infty ([0,1]) \) provare che \[ \lim_{n \to \infty} (n+1) \int_0^1 t^n f(t) \, dt =0 \quad \Longleftrightarrow \quad \lim_{r \to 1^-} \frac{1}{1-r} \int_r^1 f(t) \, dt =0. \]
La direzione "facile" dovrebbe essere \( \Longrightarrow \), ma non ho idea di come si faccia. Per funzioni \( C^1\) (o continue) sono facili entrambe le direzioni, ma se si passa ad approssimare \(f\) in \(L^p\) con una successione di funzioni regolari non si riesce a concludere. Sarei interessato ad avere un hint, o una discussione con qualcuno.
Delirium
 

Re: \( \lim_n (n+1) \int_0^1 t^n f(t) \, dt = 0 \Longleftrightarrow \lim_{r \to 1^-} \frac{1}{1-r} \int_r^1 f(t) \, dt =0\)

Messaggioda Bremen000 » 02/02/2018, 16:30

Ciao, lungi da me dire qualcosa di intelligente. Mi sono messo solo un po' a giocare con i due integrali e vorrei sapere a che conclusioni sei giunto.

Se $f$ è $C^1$: per la seconda condizione basta il teorema della media integrale (tanto $f$ è continua) e arrivo a dire che la seconda condizione equivale a $f(1)=0$. Per la prima condizione integro per parti + Beppo Levi e anche qua mi trovo che la prima condizione è equivalente a $f(1)=0$.

Se $f$ è $C^0$: per la seconda condizione tutto identico al primo caso. Per la prima condizione spezzo l'integrale da $0$ a $1/(n+1)$ e da $1/(n+1)$ in poi e anche qua arrivo a dire che la prima condizione è equivalente a $f(1)=0$.

Con $f$ in $L^p$ non c'ho nemmeno provato, ma (in $L^{\infty}$) non si può usare una versione del teorema della media un po' raffinata? Con esssup e essinf?

Oppure usare il fatto che le funzioni continue sono debolmente dense in quelle essenzialmente limitate?
"Nessuno riuscirà a cacciarci dal Paradiso che Cantor ha creato per noi." (Hilbert)
Bremen000
Advanced Member
Advanced Member
 
Messaggio: 585 di 2648
Iscritto il: 08/09/2015, 11:16

Re: \( \lim_n (n+1) \int_0^1 t^n f(t) \, dt = 0 \Longleftrightarrow \lim_{r \to 1^-} \frac{1}{1-r} \int_r^1 f(t) \, dt =0\)

Messaggioda Delirium » 02/02/2018, 17:05

Bremen000 ha scritto:[...] Se $f$ è $C^1$: per la seconda condizione basta il teorema della media integrale (tanto $f$ è continua) e arrivo a dire che la seconda condizione equivale a $f(1)=0$. Per la prima condizione integro per parti + Beppo Levi e anche qua mi trovo che la prima condizione è equivalente a $f(1)=0$.

Se $f$ è $C^0$: per la seconda condizione tutto identico al primo caso. Per la prima condizione spezzo l'integrale da $0$ a $1/(n+1)$ e da $1/(n+1)$ in poi e anche qua arrivo a dire che la prima condizione è equivalente a $f(1)=0$. [...]

Esatto.

Bremen000 ha scritto:[...] Oppure usare il fatto che le funzioni continue sono debolmente dense in quelle essenzialmente limitate? [...]

Qui avevo pensato di usare questo: se \( f \in L^\infty ([0,1]) \) allora \( f \in L^p ([0,1]) \) per ogni \( p \in [1,+\infty) \) anche; con un argomento standard di convoluzione si prova che esiste una successione di funzioni \( C^\infty\) a supporto compatto che converge ad \(f\) in qualsiasi norma \(p\); questo pero' non e' abbastanza, o comunque io non sono riuscito a farlo funzionare. Tu avevi in mente qualcos'altro? A quale topologia debole ti riferisci?
Delirium
 

Re: \( \lim_n (n+1) \int_0^1 t^n f(t) \, dt = 0 \Longleftrightarrow \lim_{r \to 1^-} \frac{1}{1-r} \int_r^1 f(t) \, dt =0\)

Messaggioda Bremen000 » 02/02/2018, 17:30

Mi riferisco al fatto che

Per ogni funzione \(f \in L^\infty ([0,1])\) esiste \(\{f_n\}_{n \in \mathbb{N}} \subset C^0([0,1])\) tale che $int_0^1f_ng d\mu \overset{\n \to \infty}{\to} \int_0^1fg d\mu$ per tutte le \(g \in L^1([0,1])\)

Quindi vedendo $L^{\infty}$ come duale di $L^1$ e usando la convergenza debole* (se così si dice in italiano...)
"Nessuno riuscirà a cacciarci dal Paradiso che Cantor ha creato per noi." (Hilbert)
Bremen000
Advanced Member
Advanced Member
 
Messaggio: 586 di 2648
Iscritto il: 08/09/2015, 11:16

Re: \( \lim_n (n+1) \int_0^1 t^n f(t) \, dt = 0 \Longleftrightarrow \lim_{r \to 1^-} \frac{1}{1-r} \int_r^1 f(t) \, dt =0\)

Messaggioda Delirium » 02/02/2018, 18:53

A questo non avevo pensato. Appena ho del tempo faccio qualche conto, grazie!
Delirium
 

Re: \( \lim_n (n+1) \int_0^1 t^n f(t) \, dt = 0 \Longleftrightarrow \lim_{r \to 1^-} \frac{1}{1-r} \int_r^1 f(t) \, dt =0\)

Messaggioda Bremen000 » 02/02/2018, 19:17

Prego! Sappimi dire cosa ti salta fuori! Anche se, riflettendo, il fatto che la convergenza non sia uniforme rispetto alle $g$ secondo me può creare qualche problema...
"Nessuno riuscirà a cacciarci dal Paradiso che Cantor ha creato per noi." (Hilbert)
Bremen000
Advanced Member
Advanced Member
 
Messaggio: 587 di 2648
Iscritto il: 08/09/2015, 11:16

Re: \( \lim_n (n+1) \int_0^1 t^n f(t) \, dt = 0 \Longleftrightarrow \lim_{r \to 1^-} \frac{1}{1-r} \int_r^1 f(t) \, dt =0\)

Messaggioda dissonance » 02/02/2018, 20:19

Bremen000 ha scritto: il fatto che la convergenza non sia uniforme rispetto alle $g$ secondo me può creare qualche problema...

Penso proprio di sì. Comunque tu voglia chiamare la tecnica per innescare un argomento di approssimazione, dovrai sempre metterci dentro una stima uniforme, altrimenti non funzionerà. La sola definizione di convergenza debole non può essere sufficiente a dimostrare nulla. Questo me lo disse quasi dieci anni fa ViciousGoblin:

viewtopic.php?p=278109#p278109
dissonance
Moderatore
Moderatore
 
Messaggio: 13584 di 27760
Iscritto il: 24/05/2008, 19:39
Località: Nomade

Re: \( \lim_n (n+1) \int_0^1 t^n f(t) \, dt = 0 \Longleftrightarrow \lim_{r \to 1^-} \frac{1}{1-r} \int_r^1 f(t) \, dt =0\)

Messaggioda dissonance » 02/02/2018, 21:47

Io farei così, ma è solo un abbozzo. Per $f\in L^\infty(0,1)$ siano
\[
T_n f=\int_0^1 (n+1)t^n f(t)\, dt,\quad R_r f = \int_0^1\frac{ [r<t<1]}{1-r}f(t)\,dt, \]
dove \([r<t<1]\) è la funzione di \(t\) che vale \(1\) se la condizione \(r<t<1\) è verificata e \(0\) altrimenti. In particolare
\[
|T_n f- R_r f|\le \|f\|_\infty \int_0^1 \left| (n+1)t^n -\frac{ [r<t<1]}{1-r}\right|\, dt, \]
e si tratta di dimostrare che l’integrale a destra può essere reso arbitrariamente piccolo scegliendo \(n\) sufficientemente grande e \(r\) sufficientemente vicino ad \(1\).

P.S.: In particolare, io eviterei un argomento di approssimazione. La ragione per questo è che lo spazio \(L^\infty\) non contiene sottospazi densi interessanti: neanche lo spazio delle funzioni continue è denso.
dissonance
Moderatore
Moderatore
 
Messaggio: 13585 di 27760
Iscritto il: 24/05/2008, 19:39
Località: Nomade

Re: \( \lim_n (n+1) \int_0^1 t^n f(t) \, dt = 0 \Longleftrightarrow \lim_{r \to 1^-} \frac{1}{1-r} \int_r^1 f(t) \, dt =0\)

Messaggioda dissonance » 04/02/2018, 20:04

dissonance ha scritto:Io farei così, ma è solo un abbozzo.[...]

Non sono più tanto sicuro che funzioni. Ho fatto due conti e mi sono accorto che la situazione è piuttosto sottile. Il procedimento che suggerisco nel mio post precedente non usa l'ipotesi che \(T_nf\to 0\) [risp. \(R_rf\to 0\)] e punta a dimostrare una cosa più forte:
\[
\tag{???} \lim_{n\to \infty} T_n f = \lim_{r\to 1^-} R_r f \quad \text{ se uno dei due limiti esiste,}
\]
ma (???) potrebbe essere falsa e di sicuro è più difficile da dimostrare.

È un esercizio un po' seccante perché il fatto che un parametro sia discreto e l'altro continuo è piuttosto fastidioso.
dissonance
Moderatore
Moderatore
 
Messaggio: 13593 di 27760
Iscritto il: 24/05/2008, 19:39
Località: Nomade

Re: \( \lim_n (n+1) \int_0^1 t^n f(t) \, dt = 0 \Longleftrightarrow \lim_{r \to 1^-} \frac{1}{1-r} \int_r^1 f(t) \, dt =0\)

Messaggioda dissonance » 04/02/2018, 21:52

Questo esercizio è essenzialmente il teorema 7 punto 2 pag. 714 di "Partial differential equations" di Evans, *seconda edizione*. Solo che nel libro la successione \((n+1)t^n\) è sostituita da \(\eta_\epsilon(x)=\frac{1}{\epsilon}\eta(\frac{x}{\epsilon})\), dove \(\eta\) ha supporto compatto. Quindi nel libro è più facile. Ma l'idea deve essere la stessa.

-----

MEGLIO: Questo è il teorema 8.15 pag.243 di Folland "Real Analysis", seconda edizione. Si può usare il teorema come scatola nera usando un cambio di variabile esponenziale (vedi sotto).


Immagine

Proposizione. Siano \(T_n\) e \(R_r\) come nel mio post precedente. Se \(f\in L^\infty(0,1)\) è tale che \(R_r f\to 0\) per \(r\to 1^-\) allora \(T_n f\to 0\).

Dimostrazione. Con il cambio di variabile \(t=e^s\) si ha
\[
T_n f=\int_{-\infty}^0 f(e^s)(n+1)e^{(n+1)s}\, ds = \int_{-\infty}^0 [f(e^s)e^s]n\phi(n s)\, ds + o(1, n\to\infty)\]
dove \(\phi(t)=e^t\mathbf 1_{t<0}\) è una funzione nonnegativa con integrale \(1\). Resta quindi da dimostrare che \(0\) è un punto di Lebesgue per la funzione \(s\mapsto |f(e^s)|e^s\). Usando lo stesso cambio di variabile \(t=e^s\) nella proprietà \(R_r f\to 0\) otteniamo che
\[
0 \leftarrow \frac{1}{1-r}\int_r^1 |f(t)|\,dt = \frac{1}{1-r}\int_{\log r}^0 |f(e^s)|e^s\, ds \ge \frac{1}{|\log r|} \int_{\log r}^0 |f(e^s)|e^s\, ds , \]
perché \(|\log r|\le 1 -r \) per \(r<1\), quindi il membro destro di questa disuguaglianza tende a \(0\) e la dimostrazione è completata.
Ultima modifica di dissonance il 05/02/2018, 00:13, modificato 1 volta in totale.
dissonance
Moderatore
Moderatore
 
Messaggio: 13594 di 27760
Iscritto il: 24/05/2008, 19:39
Località: Nomade

Prossimo

Torna a Analisi superiore

Chi c’è in linea

Visitano il forum: Nessuno e 1 ospite